PT30.S2.Q15 - during the recent economic downturn

Giant PandaGiant Panda Alum Member
edited February 2017 in Logical Reasoning 274 karma

Hi guys,

I was wondering if you wondered the same thing when you entered into this lesson question:

https://7sage.com/lesson/regulating-the-banks-na-question/
https://7sage.com/lsat_explanations/lsat-30-section-2-question-15/

Is it correct if I translate the argument into a logic as:

Tightening-->Loan Less-->Economic Downturn
Tightening
Conclusion: Economic downturn

Aside of this logical flaw, the problem is wrong due to temporal causation flaw.

Am I correct about this?

If so, I guess you can still generate answer choice if the argument is bad as a start even logically invalid?

Thanks,

Panda

Comments

  • DumbHollywoodActorDumbHollywoodActor Alum Inactive ⭐
    7468 karma

    You're using a conditional framework when this argument is clearly showing you a correlation in the premises and, then, concluding causation. See this lesson: https://7sage.com/lesson/causation/ and do that whole section, in fact) While this type of framework isn't terribly common on an NA question, it's helpful to know that blocking an alternate cause can be a necessary assumption of every causation argument drawn from correlative premises. This question gave you an easy possible alternative cause because the stimulus had already hinted to another possible cause (the economic downturn), but there were plenty more necessary assumptions for this argument. Can you think of any other necessary assumptions this argument makes?

  • SamiSami Live Member Sage 7Sage Tutor
    10795 karma

    This is why, just because you see a word "if" does not mean its a conditional argument. You really have to understand how the author is using his premise to support his conclusion.
    In our premise, we had tighter regulatory standards occur right before the banks loaned out less money which was a factor contributing to economic decline. These statements are not in dispute. What is in dispute is the conclusion which is saying that somehow the regulatory standards had something to do with banks loaning out less money; so if you don't have these standards you won't have banks loaning out less money. But in our premise, we only know that it happened before, nothing about causation (and that's why this a causal argument).

    Answer choice "A" is simply saying that a decrease in deposit money was not the cause of bank loaning out less money during downturn. Because if that was the case, we can no longer say that it was the regulatory standards that caused it. It's blocking an alternate cause.

  • Giant PandaGiant Panda Alum Member
    274 karma

    @DumbHollywoodActor said:
    it's helpful to know that blocking an alternate cause can be a necessary assumption of every causation argument drawn from correlative premises.

    fantastic. 太棒了

  • Giant PandaGiant Panda Alum Member
    274 karma
  • Giant PandaGiant Panda Alum Member
    274 karma

    Will remember it in the future. Thank you very much.

Sign In or Register to comment.